Fight Finance

Courses  Tags  Random  All  Recent  Scores

Question 160  interest only loan

You want to buy an apartment priced at $500,000. You have saved a deposit of $50,000. The bank has agreed to lend you the $450,000 as an interest only loan with a term of 30 years. The interest rate is 6% pa and is not expected to change. What will be your monthly payments?



Question 350  CFFA

Find Sidebar Corporation's Cash Flow From Assets (CFFA), also known as Free Cash Flow to the Firm (FCFF), over the year ending 30th June 2013.

Sidebar Corp
Income Statement for
year ending 30th June 2013
  $m
Sales 405
COGS 100
Depreciation 34
Rent expense 22
Interest expense 39
Taxable Income 210
Taxes at 30% 63
Net income 147
 
Sidebar Corp
Balance Sheet
as at 30th June 2013 2012
  $m $m
Cash 0 0
Inventory 70 50
Trade debtors 11 16
Rent paid in advance 4 3
PPE 700 680
Total assets 785 749
 
Trade creditors 11 19
Bond liabilities 400 390
Contributed equity 220 220
Retained profits 154 120
Total L and OE 785 749
 

 

Note: All figures are given in millions of dollars ($m).

The cash flow from assets was:



Question 383  Merton model of corporate debt, real option, option

In the Merton model of corporate debt, buying a levered company's debt is equivalent to buying the company's assets and:



Question 456  inflation, effective rate

In the 'Austin Powers' series of movies, the character Dr. Evil threatens to destroy the world unless the United Nations pays him a ransom (video 1, video 2). Dr. Evil makes the threat on two separate occasions:

  • In 1969 he demands a ransom of $1 million (=10^6), and again;
  • In 1997 he demands a ransom of $100 billion (=10^11).

If Dr. Evil's demands are equivalent in real terms, in other words $1 million will buy the same basket of goods in 1969 as $100 billion would in 1997, what was the implied inflation rate over the 28 years from 1969 to 1997?

The answer choices below are given as effective annual rates:


Question 547  PE ratio, Multiples valuation, DDM, income and capital returns, no explanation

A firm pays out all of its earnings as dividends. Because of this, the firm has no real growth in earnings, dividends or stock price since there is no re-investment back into the firm to buy new assets and make higher earnings. The dividend discount model is suitable to value this company.

The firm's revenues and costs are expected to increase by inflation in the foreseeable future. The firm has no debt. It operates in the services industry and has few physical assets so there is negligible depreciation expense and negligible net working capital required.

Which of the following statements about this firm's PE ratio is NOT correct? The PE ratio should:

Note: The inverse of x is 1/x.



Question 600  foreign exchange rate

A Chinese man wishes to convert AUD 1 million into Chinese Renminbi (RMB, also called the Yuan (CNY)). The exchange rate is 6.35 RMB per USD, and 0.72 USD per AUD. How much is the AUD 1 million worth in RMB?



Question 694  utility

Which of the below statements about utility is NOT generally accepted by economists? Most people are thought to:



Question 834  option, delta, theta, gamma, standard deviation, Black-Scholes-Merton option pricing

Which of the following statements about an option (either a call or put) and its underlying stock is NOT correct?

European Call Option
on a non-dividend paying stock
Description Symbol Quantity
Spot price ($) ##S_0## 20
Strike price ($) ##K_T## 18
Risk free cont. comp. rate (pa) ##r## 0.05
Standard deviation of the stock's cont. comp. returns (pa) ##\sigma## 0.3
Option maturity (years) ##T## 1
Call option price ($) ##c_0## 3.939488
Delta ##\Delta = N[d_1]## 0.747891
##N[d_2]## ##N[d_2]## 0.643514
Gamma ##\Gamma## 0.053199
Theta ($/year) ##\Theta = \partial c / \partial T## 1.566433
 

 



Question 943  dividend date, ex dividend date

On which date would the stock price increase if the dividend and earnings are higher than expected?



Question 992  inflation, real and nominal returns and cash flows

You currently have $100 in the bank which pays a 10% pa interest rate.

Oranges currently cost $1 each at the shop and inflation is 5% pa which is the expected growth rate in the orange price.

This information is summarised in the table below, with some parts missing that correspond to the answer options. All rates are given as effective annual rates. Note that when payments are not specified as real, as in this question, they're conventionally assumed to be nominal.

Wealth in Dollars and Oranges
Time (year) Bank account wealth ($) Orange price ($) Wealth in oranges
0 100 1 100
1 110 1.05 (a)
2 (b) (c) (d)
 

 

Which of the following statements is NOT correct? Your: